PT71-S1-Q22 Help (Predatory Pricing) Forum

Prepare for the LSAT or discuss it with others in this forum.
Post Reply
akechi

New
Posts: 74
Joined: Fri Jun 21, 2013 11:38 am

PT71-S1-Q22 Help (Predatory Pricing)

Post by akechi » Mon Sep 22, 2014 8:17 pm

Hi All,

I need some help wrapping my head around this problem. I am trying to figure out where I took a wrong turn while answering this question.

I will give a quick run-down of thought process so someone can help me identify where my process led me astray.

Task:
Find an answer choice that must be true in order for the conclusion to be possibly true. We know the argument will be flawed in some way. We know the premises won't fully support the conclusion - find the gap(s) / flaw(s).

Conclusion:
Predatory pricing should be acceptable

Why? Premise:
The mere threat of competition will keep prices down (at least to the extent of not raising beyond unreasonable levels).

Analysis:
My initial gut reaction was: "who cares if prices are down, the author hasn't established an exclusive relationship between prices and acceptable-ness. What about all these other factors that could be relevant to the acceptable-ness of predatory pricing. For example, the prices might be kept down, but what about the the potential moral issue of driving a family-owned business out of business. "

Going into the answer choices, I was looking for an answer that would expose the LSAT 'arrogance' that the one given cause / reason is the only one.

Answer Choices:
I quickly eliminated (A) - (C)
I was down to (D) and (E), and in a rush, I chose (D) because it superficially looked like what I was hunting for.

But coming back to it, is (D) wrong because 1) it doesn't really relate the pricing to the acceptable-ness of predatory pricing (it only seems to reinforce the strength of the premise), 2) the negation of it actually strengthens the argument.

(E) is correct, but I had trouble wrapping my head around exactly what it was trying to save because of the double-negative. Is it safe to convert double negatives in a positive frame by canceling out the two negative terms? In this case, I would be inclined to re-word the statement: "any pricing practice that does result in reasonable prices should be acceptable".

User avatar
walterwhite

Bronze
Posts: 178
Joined: Thu Jul 18, 2013 8:31 pm

Re: PT71-S1-Q22 Help (Predatory Pricing)

Post by walterwhite » Mon Sep 22, 2014 8:25 pm

I made the exact same mistake. During blind review I was like wait a minute, there could be a million things that keep companies from raising prices, and the argument would still hold. So D is definitely out. I think this was a question where you just need to find the conclusion in the argument, which is that "The practice should clearly be acceptable." If you focus on that word "acceptable," then you see that E is the strongest answer choice.

User avatar
dontdoitkid

Bronze
Posts: 191
Joined: Thu Jan 02, 2014 8:02 pm

Re: PT71-S1-Q22 Help (Predatory Pricing)

Post by dontdoitkid » Mon Sep 22, 2014 8:35 pm

It can definitely be "safe", as long as you try to keep what the statement is actually intending to mean at the forefront. You wrote that you negated it to say: "Any pricing practice that does result in reasonable prices should be acceptable". Think about how this applies to the argument. We are trying to connect the following pieces of the argument:

1. Sometimes there is public outcry against predatory pricing.

2. It should be acceptable however, since the the threat of more competition will prevent prices from reaching unreasonable levels.

So if there's public outcry against it, why should it be acceptable? If we were to apply your reversal - "Any pricing practice that does result in reasonable prices should be acceptable" we will not actually have the assumption we need. The predatory pricing doesn't necessarily result in reasonable prices... it just prevents unreasonable prices. They seem like opposites, but remember to think in terms of logical opposites on the LSAT (The opposite of high isn't low, it's not-high).

Now what about option E? It connects the initial premise (Predatory Pricing results in a condition where merely the threat of renewed competition prevents prices being raised to unreasonable levels) with the conclusion (the practice should clearly be acceptable).

Since E says any pricing pracitce that doesn't result in unreasonable prices is acceptable, it forms a perfect link between the two.

Hope that helps!

silvermanlsatprep

New
Posts: 3
Joined: Sat Sep 13, 2014 11:36 pm

Re: PT71-S1-Q22 Help (Predatory Pricing)

Post by silvermanlsatprep » Mon Sep 22, 2014 8:44 pm

Let's carefully examine the argument.

First they define "predatory pricing" for us, and a claim is made that occasionally there is a public outcry against it. A conclusion is then drawn that even though there is an outcry, the pricing is acceptable because even though the pricing might drive out competition, businesses engaging in the pricing will still keep their prices reasonable because they will fear future competition.

But let's assume that that predatory pricing can be deemed unacceptable even if the pricing does not lead to unreasonable prices. If that were true, then the conclusion (that the pricing is definitely acceptable) would no longer be valid, because the only reason we were given as to why the pricing was acceptable is because the pricing would not lead to unreasonable prices.

So, therefore, the conclusion (that predatory pricing is acceptable) requires an assumption that any pricing practice that does not result in unreasonable prices is acceptable.

--Sean

User avatar
Christine (MLSAT)

Bronze
Posts: 357
Joined: Fri Nov 22, 2013 3:41 pm

Re: PT71-S1-Q22 Help (Predatory Pricing)

Post by Christine (MLSAT) » Mon Sep 22, 2014 9:19 pm

Hey Akechi!

Your initial breakdown work and pre-phrasing were pretty spot on. You might have saved yourself some trouble if you had done one of two things: 1) focus on the new item in the conclusion or 2) articulate slightly more distinctly what the gap is.

You clearly picked up on the need to connect the price issue to acceptability. Only (E) even raises the issue of acceptability!

Now, what I mean by articulating the gap - sometimes, just an awareness that we need to connect [prices] to [acceptability] is enough to get us through the answers. However, we can go further. Many, many assumptions on the LSAT follow the essential format of "If [premise], then [conclusion]". Following that format, you could have articulated the connection as:

If [prices prevented from being unreasonable], then [acceptable].

If you had done this, the double negative might not have bothered you so much because the same double negative pops up in our premise! If prices are prevented from being unreasonable, then they must be reasonable.

You are totally allowed to make a double negative into a positive IF it really is true that it means the same thing. If someone, annoyingly, said "the amount was not a non-trivial matter", that means it WAS trivial. However, there are exceptions to this idea you'd have to think more carefully about - if I say that I'm not unlucky, that may not really mean that I'm lucky. It's possible to be neither lucky nor unlucky - just neutral.

Here, though, I'm comfortable that things can only be reasonable or unreasonable - there's no in between. (But notice how the LSAT carefully, almost paranoidly, avoids the issue by maintaining the double negative expression in both the premise and the answer choice. Hmmmm.... :P )

(D) is tempting primarily because it is the logical reversal of the conditional given in the premise. So, it's in scope, has a lot of the right terms, etc. But we don't need it. (Also, it doesn't connect to anything in the conclusion!)

Also, I just want to say that I'm so glad to see how far you've come over the last few months. You're gonna knock 'em dead Saturday. :mrgreen:

Want to continue reading?

Register now to search topics and post comments!

Absolutely FREE!


User avatar
Louis1127

Silver
Posts: 817
Joined: Thu Jun 27, 2013 9:12 pm

Re: PT71-S1-Q22 Help (Predatory Pricing)

Post by Louis1127 » Tue Sep 23, 2014 9:27 am

Could someone comment about why the broadness of (E) is ok in this situation?

I thought to myself, "Well, this would mean that selling children would be acceptable as long as the price of them is low, but that's absurd and we don't need that to be true for this argument to work".

akechi

New
Posts: 74
Joined: Fri Jun 21, 2013 11:38 am

Re: PT71-S1-Q22 Help (Predatory Pricing)

Post by akechi » Tue Sep 23, 2014 3:47 pm

Special thanks for the responses Silverman and Christine!

Christine:
You are spot on, per usual, about how to refine my approach. I definitely could have benefited from having a more precise / clean pre-phrase as opposed to my abstract / jumbled pre-phrase.

I am working to keep my characterization of flaws and pre-phrases as simple and intuitive as possible. I often find myself wasting precious time by taking my pre-phrase and trying to produce real-life examples that would fit the pre-phrase. Silverman / your approach would have sufficed, I didn't need to think about the moral issues of driving a family owned business out of town.

+1 to the idea of focusing on new elements in the conclusion. I should have recognized that (D) actually is a common trap answer for Assumption Family questions, in that it 1) reverses the logic 2) is related to the reasoning, but not exactly what we need 3) draws on familiar elements but does not connect the right ideas.

By the way, I still use the "NUH-UH" mentally when working with Assumption Family questions, it reminds me to keep things simple. :D

Want to continue reading?

Register for access!

Did I mention it was FREE ?


Post Reply

Return to “LSAT Prep and Discussion Forum”